LSAT and Law School Admissions Forum

Get expert LSAT preparation and law school admissions advice from PowerScore Test Preparation.

 moshei24
  • Posts: 465
  • Joined: Mar 20, 2012
|
#5698
What question type is this question? I would think that it's MustX, but most of the choices don't seem to have to be true based on the stimulus. It seems more like which of the following can't the stimulus help to prove is true. It seems more like a StrengthenX.

I think this is the second time I've seen a stem like this.

Thanks!
 Jon Denning
PowerScore Staff
  • PowerScore Staff
  • Posts: 904
  • Joined: Apr 11, 2011
|
#5897
Hey moshe - from a categorization standpoint that's a Must Be True Except (MustX) question, where the four incorrect answers follow most logically/with the highest likelihood from the stimulus. However, from a purely operational standpoint it falls within that category a bit more closely to the idea of Most Strongly Supported rather than the more black-and-white/absolute Must Be True. But still a Must Except, and you can evaluate the answers based on how well supported they are from the information in the stimulus, eliminating those that are supported most convincingly.
 moshei24
  • Posts: 465
  • Joined: Mar 20, 2012
|
#5904
So I'm basically looking for the answer that I can't find any support for in the stimulus? If the answer choice has any support, it would be wrong? That's what this question seemed like to me...
 Steve Stein
PowerScore Staff
  • PowerScore Staff
  • Posts: 1153
  • Joined: Apr 11, 2011
|
#5906
Basically--you're looking for the answer choice that has no support, direct or indirect, in the information from the stimulus.

I hope that's helpful!

~Steve
 moshei24
  • Posts: 465
  • Joined: Mar 20, 2012
|
#5907
Gotcha- it's different from a MustX; I'm looking for the choice that I can't argue for based on the stimulus, correct?

Thank you!
 r miller
PowerScore Staff
  • PowerScore Staff
  • Posts: 10
  • Joined: Aug 23, 2012
|
#5910
haven't had an opportunity to look at the question, but, speaking out of my ____, and generally about the question type, i would comment that this is a bit of a specialized category of must be true question. if you look at the lesson, the last example of the question stems gives you an example of one of these. i find a lot of students have trouble with this, because it isn't exactly the same as an absolute, cut and dried, find a firm conclusion, question stem and answer choice.

i find it helps to explain it as follows. but, one must be careful, as this explanation suggests a causal idea, which is not what we're looking for here.
in any event, the idea behind the question is that the discussion in the stimulus would most likely lead one (avoid causal suggestion here) to what next step in the reasoning? or, where is the speaker in the stimulus heading, what would most appropriately follow from what is being discussed. (although one cannot be sure, so it requires a review, as always, of all answers, and allows the lsat to give you a tougher question!)

you should see that this is a bit more difficult concept, because it places you in the mind of the speaker (or author), and forces you to assume a little bit as to where the discussion is headed, always a concern. reminds a little bit of the function question in reading comprehension.

well, we strayed into the philosophical there. sorry about that. better stick with jon and steve and get a good, direct, answer, lol!

hope that helps!

tx

Get the most out of your LSAT Prep Plus subscription.

Analyze and track your performance with our Testing and Analytics Package.